Prove or disprove: If $A$ is $ntimes n$ and $exists;min BbbN:;A^m=I_n$, then $A$ is invertible. The Next CEO of Stack OverflowDeterminants of matrices; Does the equation $det M=det(I_n-M)$ have a solution?If $A^2+2A+I_n=O_n$ then $A$ is invertibleDeterminant of a $2 times 2$ block matrixprove that if a square matrix $A$ is invertible then $AA^T$ is invertible.Matrices such that $M^2+M^T=I_n$ are invertibleIf a matrix is row equivalent to some invertible matrix then it is invertibleProve that $A+I_n$ is invertible, where $leftlVert ArightrVert<1$Is it true that $m=nimplies A$ is invertible, for an $mtimes n$ matrix satisfying $(AA^T)^r=I$Help with proof or counterexample: $A^3=0 implies I_n+A$ is invertibleThe matrix $I_n - v^t x$ is invertible when $langle v, x rangle neq 1$

Why were Madagascar and New Zealand discovered so late?

Too much space between section and text in a twocolumn document

Trouble understanding the speech of overseas colleagues

If the heap is initialized for security, then why is the stack uninitialized?

How easy is it to start Magic from scratch?

How to make a software documentation "officially" citable?

How to use tikz in fbox?

Is it okay to store user locations?

Return the Closest Prime Number

How long to clear the 'suck zone' of a turbofan after start is initiated?

Describing a person. What needs to be mentioned?

Go Pregnant or Go Home

Where to find order of arguments for default functions

What is the difference between "behavior" and "behaviour"?

What do "high sea" and "carry" mean in this sentence?

Apart from "berlinern", do any other German dialects have a corresponding verb?

Opposite of a diet

Whats the best way to handle refactoring a big file?

Why didn't Khan get resurrected in the Genesis Explosion?

What is the purpose of the Evocation wizard's Potent Cantrip feature?

Unreliable Magic - Is it worth it?

Can a caster that cast Polymorph on themselves stop concentrating at any point even if their Int is low?

How to write the block matrix in LaTex?

Does the Brexit deal have to be agreed by both Houses?



Prove or disprove: If $A$ is $ntimes n$ and $exists;min BbbN:;A^m=I_n$, then $A$ is invertible.



The Next CEO of Stack OverflowDeterminants of matrices; Does the equation $det M=det(I_n-M)$ have a solution?If $A^2+2A+I_n=O_n$ then $A$ is invertibleDeterminant of a $2 times 2$ block matrixprove that if a square matrix $A$ is invertible then $AA^T$ is invertible.Matrices such that $M^2+M^T=I_n$ are invertibleIf a matrix is row equivalent to some invertible matrix then it is invertibleProve that $A+I_n$ is invertible, where $leftlVert ArightrVert<1$Is it true that $m=nimplies A$ is invertible, for an $mtimes n$ matrix satisfying $(AA^T)^r=I$Help with proof or counterexample: $A^3=0 implies I_n+A$ is invertibleThe matrix $I_n - v^t x$ is invertible when $langle v, x rangle neq 1$










3












$begingroup$


Is this statement true? If $A$ is an $ntimes n$ matrix and $A^m=I_n$ for some $min BbbN$, then $A$ is invertible.



My trial



Let $nin BbbN$ be fixed. Then, $$[det(A)]^m=det(A^m)=I_n=1.$$
Hence, $$det(A)=1neq 0.$$
Thus, $A$ is invertible since $det(A)neq 0.$. I'm I right or is there a counter-example?










share|cite|improve this question











$endgroup$







  • 2




    $begingroup$
    Looks good. Alternatively, $A(A^m-1)=I=(A^m-1)A$ so $A^-1=A^m-1$.
    $endgroup$
    – Chrystomath
    Mar 18 at 11:32
















3












$begingroup$


Is this statement true? If $A$ is an $ntimes n$ matrix and $A^m=I_n$ for some $min BbbN$, then $A$ is invertible.



My trial



Let $nin BbbN$ be fixed. Then, $$[det(A)]^m=det(A^m)=I_n=1.$$
Hence, $$det(A)=1neq 0.$$
Thus, $A$ is invertible since $det(A)neq 0.$. I'm I right or is there a counter-example?










share|cite|improve this question











$endgroup$







  • 2




    $begingroup$
    Looks good. Alternatively, $A(A^m-1)=I=(A^m-1)A$ so $A^-1=A^m-1$.
    $endgroup$
    – Chrystomath
    Mar 18 at 11:32














3












3








3


1



$begingroup$


Is this statement true? If $A$ is an $ntimes n$ matrix and $A^m=I_n$ for some $min BbbN$, then $A$ is invertible.



My trial



Let $nin BbbN$ be fixed. Then, $$[det(A)]^m=det(A^m)=I_n=1.$$
Hence, $$det(A)=1neq 0.$$
Thus, $A$ is invertible since $det(A)neq 0.$. I'm I right or is there a counter-example?










share|cite|improve this question











$endgroup$




Is this statement true? If $A$ is an $ntimes n$ matrix and $A^m=I_n$ for some $min BbbN$, then $A$ is invertible.



My trial



Let $nin BbbN$ be fixed. Then, $$[det(A)]^m=det(A^m)=I_n=1.$$
Hence, $$det(A)=1neq 0.$$
Thus, $A$ is invertible since $det(A)neq 0.$. I'm I right or is there a counter-example?







matrices algebra-precalculus matrix-equations






share|cite|improve this question















share|cite|improve this question













share|cite|improve this question




share|cite|improve this question








edited Mar 18 at 11:34







Omojola Micheal

















asked Mar 18 at 11:29









Omojola MichealOmojola Micheal

1,999424




1,999424







  • 2




    $begingroup$
    Looks good. Alternatively, $A(A^m-1)=I=(A^m-1)A$ so $A^-1=A^m-1$.
    $endgroup$
    – Chrystomath
    Mar 18 at 11:32













  • 2




    $begingroup$
    Looks good. Alternatively, $A(A^m-1)=I=(A^m-1)A$ so $A^-1=A^m-1$.
    $endgroup$
    – Chrystomath
    Mar 18 at 11:32








2




2




$begingroup$
Looks good. Alternatively, $A(A^m-1)=I=(A^m-1)A$ so $A^-1=A^m-1$.
$endgroup$
– Chrystomath
Mar 18 at 11:32





$begingroup$
Looks good. Alternatively, $A(A^m-1)=I=(A^m-1)A$ so $A^-1=A^m-1$.
$endgroup$
– Chrystomath
Mar 18 at 11:32











3 Answers
3






active

oldest

votes


















4












$begingroup$

The right conclusion is $det(A) ne 0$, hence it is invertible.



Notice that we can't conclude that $det(A)=1$. After all, it can take value $-1$.






share|cite|improve this answer









$endgroup$












  • $begingroup$
    Sorry, I'll edit!
    $endgroup$
    – Omojola Micheal
    Mar 18 at 11:33










  • $begingroup$
    Thanks for the prompt reply.
    $endgroup$
    – Omojola Micheal
    Mar 18 at 11:34


















6












$begingroup$

I think it is easier to see it this way: what is $AA^m-1 = A^m-1A$?






share|cite|improve this answer









$endgroup$




















    2












    $begingroup$

    It's invertible since it has a inverse, $A^m-1$ that is. By the way, $det(A)$ could also be -1.






    share|cite|improve this answer









    $endgroup$













      Your Answer





      StackExchange.ifUsing("editor", function ()
      return StackExchange.using("mathjaxEditing", function ()
      StackExchange.MarkdownEditor.creationCallbacks.add(function (editor, postfix)
      StackExchange.mathjaxEditing.prepareWmdForMathJax(editor, postfix, [["$", "$"], ["\\(","\\)"]]);
      );
      );
      , "mathjax-editing");

      StackExchange.ready(function()
      var channelOptions =
      tags: "".split(" "),
      id: "69"
      ;
      initTagRenderer("".split(" "), "".split(" "), channelOptions);

      StackExchange.using("externalEditor", function()
      // Have to fire editor after snippets, if snippets enabled
      if (StackExchange.settings.snippets.snippetsEnabled)
      StackExchange.using("snippets", function()
      createEditor();
      );

      else
      createEditor();

      );

      function createEditor()
      StackExchange.prepareEditor(
      heartbeatType: 'answer',
      autoActivateHeartbeat: false,
      convertImagesToLinks: true,
      noModals: true,
      showLowRepImageUploadWarning: true,
      reputationToPostImages: 10,
      bindNavPrevention: true,
      postfix: "",
      imageUploader:
      brandingHtml: "Powered by u003ca class="icon-imgur-white" href="https://imgur.com/"u003eu003c/au003e",
      contentPolicyHtml: "User contributions licensed under u003ca href="https://creativecommons.org/licenses/by-sa/3.0/"u003ecc by-sa 3.0 with attribution requiredu003c/au003e u003ca href="https://stackoverflow.com/legal/content-policy"u003e(content policy)u003c/au003e",
      allowUrls: true
      ,
      noCode: true, onDemand: true,
      discardSelector: ".discard-answer"
      ,immediatelyShowMarkdownHelp:true
      );



      );













      draft saved

      draft discarded


















      StackExchange.ready(
      function ()
      StackExchange.openid.initPostLogin('.new-post-login', 'https%3a%2f%2fmath.stackexchange.com%2fquestions%2f3152675%2fprove-or-disprove-if-a-is-n-times-n-and-exists-m-in-bbbn-am-i-n%23new-answer', 'question_page');

      );

      Post as a guest















      Required, but never shown

























      3 Answers
      3






      active

      oldest

      votes








      3 Answers
      3






      active

      oldest

      votes









      active

      oldest

      votes






      active

      oldest

      votes









      4












      $begingroup$

      The right conclusion is $det(A) ne 0$, hence it is invertible.



      Notice that we can't conclude that $det(A)=1$. After all, it can take value $-1$.






      share|cite|improve this answer









      $endgroup$












      • $begingroup$
        Sorry, I'll edit!
        $endgroup$
        – Omojola Micheal
        Mar 18 at 11:33










      • $begingroup$
        Thanks for the prompt reply.
        $endgroup$
        – Omojola Micheal
        Mar 18 at 11:34















      4












      $begingroup$

      The right conclusion is $det(A) ne 0$, hence it is invertible.



      Notice that we can't conclude that $det(A)=1$. After all, it can take value $-1$.






      share|cite|improve this answer









      $endgroup$












      • $begingroup$
        Sorry, I'll edit!
        $endgroup$
        – Omojola Micheal
        Mar 18 at 11:33










      • $begingroup$
        Thanks for the prompt reply.
        $endgroup$
        – Omojola Micheal
        Mar 18 at 11:34













      4












      4








      4





      $begingroup$

      The right conclusion is $det(A) ne 0$, hence it is invertible.



      Notice that we can't conclude that $det(A)=1$. After all, it can take value $-1$.






      share|cite|improve this answer









      $endgroup$



      The right conclusion is $det(A) ne 0$, hence it is invertible.



      Notice that we can't conclude that $det(A)=1$. After all, it can take value $-1$.







      share|cite|improve this answer












      share|cite|improve this answer



      share|cite|improve this answer










      answered Mar 18 at 11:32









      Siong Thye GohSiong Thye Goh

      103k1468119




      103k1468119











      • $begingroup$
        Sorry, I'll edit!
        $endgroup$
        – Omojola Micheal
        Mar 18 at 11:33










      • $begingroup$
        Thanks for the prompt reply.
        $endgroup$
        – Omojola Micheal
        Mar 18 at 11:34
















      • $begingroup$
        Sorry, I'll edit!
        $endgroup$
        – Omojola Micheal
        Mar 18 at 11:33










      • $begingroup$
        Thanks for the prompt reply.
        $endgroup$
        – Omojola Micheal
        Mar 18 at 11:34















      $begingroup$
      Sorry, I'll edit!
      $endgroup$
      – Omojola Micheal
      Mar 18 at 11:33




      $begingroup$
      Sorry, I'll edit!
      $endgroup$
      – Omojola Micheal
      Mar 18 at 11:33












      $begingroup$
      Thanks for the prompt reply.
      $endgroup$
      – Omojola Micheal
      Mar 18 at 11:34




      $begingroup$
      Thanks for the prompt reply.
      $endgroup$
      – Omojola Micheal
      Mar 18 at 11:34











      6












      $begingroup$

      I think it is easier to see it this way: what is $AA^m-1 = A^m-1A$?






      share|cite|improve this answer









      $endgroup$

















        6












        $begingroup$

        I think it is easier to see it this way: what is $AA^m-1 = A^m-1A$?






        share|cite|improve this answer









        $endgroup$















          6












          6








          6





          $begingroup$

          I think it is easier to see it this way: what is $AA^m-1 = A^m-1A$?






          share|cite|improve this answer









          $endgroup$



          I think it is easier to see it this way: what is $AA^m-1 = A^m-1A$?







          share|cite|improve this answer












          share|cite|improve this answer



          share|cite|improve this answer










          answered Mar 18 at 11:33









          MariahMariah

          2,1431718




          2,1431718





















              2












              $begingroup$

              It's invertible since it has a inverse, $A^m-1$ that is. By the way, $det(A)$ could also be -1.






              share|cite|improve this answer









              $endgroup$

















                2












                $begingroup$

                It's invertible since it has a inverse, $A^m-1$ that is. By the way, $det(A)$ could also be -1.






                share|cite|improve this answer









                $endgroup$















                  2












                  2








                  2





                  $begingroup$

                  It's invertible since it has a inverse, $A^m-1$ that is. By the way, $det(A)$ could also be -1.






                  share|cite|improve this answer









                  $endgroup$



                  It's invertible since it has a inverse, $A^m-1$ that is. By the way, $det(A)$ could also be -1.







                  share|cite|improve this answer












                  share|cite|improve this answer



                  share|cite|improve this answer










                  answered Mar 18 at 11:46









                  Sander KortewegSander Korteweg

                  566




                  566



























                      draft saved

                      draft discarded
















































                      Thanks for contributing an answer to Mathematics Stack Exchange!


                      • Please be sure to answer the question. Provide details and share your research!

                      But avoid


                      • Asking for help, clarification, or responding to other answers.

                      • Making statements based on opinion; back them up with references or personal experience.

                      Use MathJax to format equations. MathJax reference.


                      To learn more, see our tips on writing great answers.




                      draft saved


                      draft discarded














                      StackExchange.ready(
                      function ()
                      StackExchange.openid.initPostLogin('.new-post-login', 'https%3a%2f%2fmath.stackexchange.com%2fquestions%2f3152675%2fprove-or-disprove-if-a-is-n-times-n-and-exists-m-in-bbbn-am-i-n%23new-answer', 'question_page');

                      );

                      Post as a guest















                      Required, but never shown





















































                      Required, but never shown














                      Required, but never shown












                      Required, but never shown







                      Required, but never shown

































                      Required, but never shown














                      Required, but never shown












                      Required, but never shown







                      Required, but never shown







                      Popular posts from this blog

                      Lowndes Grove History Architecture References Navigation menu32°48′6″N 79°57′58″W / 32.80167°N 79.96611°W / 32.80167; -79.9661132°48′6″N 79°57′58″W / 32.80167°N 79.96611°W / 32.80167; -79.9661178002500"National Register Information System"Historic houses of South Carolina"Lowndes Grove""+32° 48' 6.00", −79° 57' 58.00""Lowndes Grove, Charleston County (260 St. Margaret St., Charleston)""Lowndes Grove"The Charleston ExpositionIt Happened in South Carolina"Lowndes Grove (House), Saint Margaret Street & Sixth Avenue, Charleston, Charleston County, SC(Photographs)"Plantations of the Carolina Low Countrye

                      random experiment with two different functions on unit interval Announcing the arrival of Valued Associate #679: Cesar Manara Planned maintenance scheduled April 23, 2019 at 00:00UTC (8:00pm US/Eastern)Random variable and probability space notionsRandom Walk with EdgesFinding functions where the increase over a random interval is Poisson distributedNumber of days until dayCan an observed event in fact be of zero probability?Unit random processmodels of coins and uniform distributionHow to get the number of successes given $n$ trials , probability $P$ and a random variable $X$Absorbing Markov chain in a computer. Is “almost every” turned into always convergence in computer executions?Stopped random walk is not uniformly integrable

                      How should I support this large drywall patch? Planned maintenance scheduled April 23, 2019 at 00:00UTC (8:00pm US/Eastern) Announcing the arrival of Valued Associate #679: Cesar Manara Unicorn Meta Zoo #1: Why another podcast?How do I cover large gaps in drywall?How do I keep drywall around a patch from crumbling?Can I glue a second layer of drywall?How to patch long strip on drywall?Large drywall patch: how to avoid bulging seams?Drywall Mesh Patch vs. Bulge? To remove or not to remove?How to fix this drywall job?Prep drywall before backsplashWhat's the best way to fix this horrible drywall patch job?Drywall patching using 3M Patch Plus Primer